Multivariable Calculus, 7th Edition

Published by Brooks Cole
ISBN 10: 0-53849-787-4
ISBN 13: 978-0-53849-787-9

Chapter 14 - Partial Derivatives - 14.5 Exercises - Page 955: 38

Answer

$8160~\pi \dfrac{in^3}{s}$

Work Step by Step

Apply the chain rule: $\dfrac{dV}{dt}=(\dfrac{\partial V}{\partial r})(\dfrac{dr}{ dt})+(\dfrac{\partial V}{\partial h})(\dfrac{dh}{dt})$ Now, $\dfrac{\partial V}{ \partial r} \approx \dfrac{\partial }{ \partial r} (\dfrac{1}{3} \pi r^2 h)=\dfrac{2}{3}\pi r^2 h$ and $\dfrac{\partial V}{ \partial h} \approx \dfrac{\partial }{ \partial h} (\dfrac{1}{3} \pi r^2 h)=\dfrac{1}{3}\pi r^2 $ Thus, we have $\dfrac{dV}{dt}=(11200 \pi)(1.8)+(4800 \pi)(-2.5)= 8160 \pi \dfrac{in^3}{s}$
Update this answer!

You can help us out by revising, improving and updating this answer.

Update this answer

After you claim an answer you’ll have 24 hours to send in a draft. An editor will review the submission and either publish your submission or provide feedback.